0% found this document useful (0 votes)
149 views7 pages

Prime Numbers: Endless and Intriguing

1) Euclid proved the infinitude of primes using the fact that any integer n greater than 1 has a prime factor. He showed that for any finite list of primes, one can construct a number with a prime factor not in the list. 2) Euler provided an alternative proof using unique factorization and the divergence of the harmonic series. By writing 1/(1-1/p) as a geometric series for each prime p, he showed their product equals the harmonic series, proving there must be infinitely many terms (and thus primes). 3) Both proofs rely on fundamental properties of numbers and have interesting consequences, such as a lower bound on the prime counting function and the divergence of the sum of reciprocals

Uploaded by

Che Tyson
Copyright
© © All Rights Reserved
We take content rights seriously. If you suspect this is your content, claim it here.
Available Formats
Download as PDF, TXT or read online on Scribd
0% found this document useful (0 votes)
149 views7 pages

Prime Numbers: Endless and Intriguing

1) Euclid proved the infinitude of primes using the fact that any integer n greater than 1 has a prime factor. He showed that for any finite list of primes, one can construct a number with a prime factor not in the list. 2) Euler provided an alternative proof using unique factorization and the divergence of the harmonic series. By writing 1/(1-1/p) as a geometric series for each prime p, he showed their product equals the harmonic series, proving there must be infinitely many terms (and thus primes). 3) Both proofs rely on fundamental properties of numbers and have interesting consequences, such as a lower bound on the prime counting function and the divergence of the sum of reciprocals

Uploaded by

Che Tyson
Copyright
© © All Rights Reserved
We take content rights seriously. If you suspect this is your content, claim it here.
Available Formats
Download as PDF, TXT or read online on Scribd
You are on page 1/ 7

THE INFINITUDE OF THE PRIMES

KEITH CONRAD

1. Introduction
The sequence of prime numbers
2, 3, 5, 7, 11, 13, 17, 19, 23, 29, 31, 37, 41, 43, 47, 53, 59, . . . , 1873, 1877, 1879, 1889, 1901, . . .
never ends. This fact has many different proofs. We’ll discuss three of them, due to Euclid,
Euler, and Erdős and some of their consequences. More proofs are in [10, Chap. 1].

2. Euclid’s proof
The standard proof of the infinitude of the primes is attributed to Euclid and uses the
fact that all integers greater than 1 have a prime factor.
Lemma 2.1. Every integer greater than 1 has a prime factor.
Proof. We argue by (strong) induction that each integer n > 1 has a prime factor. For the
base case n = 2, 2 is prime and is a factor of itself.
Now assume n > 2 all integers greater than 1 and less than n have a prime factor. To
show n has a prime factor, we take cases.
Case 1: n is prime.
Since n is a factor of itself, n has a prime factor when n is prime.
Case 2: n is not prime.
Since n is not prime, it has a factorization n = ab where 1 < a, b < n. Then by the
strong inductive hypothesis, a has a prime factor, say p. Since p | a and a | n, also p | n
and thus n has prime factor p. 
Theorem 2.2. There are infinitely many primes.
Proof. (Euclid) To show there are infinitely many primes, we’ll show that every finite list
of primes is missing a prime number, so the list of all primes can’t be finite.
To begin, there are prime numbers such as 2. Suppose p1 , . . . , pr is a finite list of prime
numbers. We want to show this is not the full list of the primes. Consider the number
N = p1 · · · pr + 1.
Since N > 1, it has a prime factor p by Lemma 2.1. The prime p can’t be any of p1 , . . . , pr
since N has remainder 1 when divided by each pi . Therefore p is a prime not on our list,
so the set of primes can’t be finite. 
Some people misunderstand this proof to be saying that if p1 , . . . , pr are prime then
p1 · · · pr + 1 is prime. That is not generally true. It starts out looking correct: 2 + 1 is
prime, 2 · 3 + 1 = 7 is prime, 2 · 3 · 5 + 1 = 31 is prime, 2 · 3 · 5 · 7 + 1 = 211 is prime, and
2 · 3 · 5 · 7 · 11 + 1 = 2311 is prime, but
2 · 3 · 5 · 7 · 11 · 13 + 1 = 30031 = 59 · 509
1
2 KEITH CONRAD

is not prime. (More simply, 2 · 7 + 1 = 15 and 3 · 5 + 1 = 16 are not prime.) Euclid’s proof
tells us that we can always find a prime outside of a finite list of primes p1 , . . . , pr by using
a prime factor of p1 · · · pr + 1, not by using p1 · · · pr + 1 itself.
Let’s try to use Euclid’s proof as a method of finding new primes, by taking the smallest
prime factor of p1 · · · pr + 1 at each step as a new prime to add to our list:
• 2 is prime and 2 + 1 = 3 is prime,
• 2 · 3 + 1 = 7 is prime,
• 2 · 3 · 7 + 1 = 43 is prime,
• 2 · 3 · 7 · 43 + 1 = 1807 = 13 · 139,
• 2 · 3 · 7 · 43 · 13 + 1 = 23479 = 53 · 443,
• 2 · 3 · 7 · 43 · 13 · 53 + 1 = 1244335 = 5 · 248867.
So far this list of primes in the order they appear is 2, 3, 7, 43, 13, 53, 5. This way of creating
new primes was introduced by Mullin [8] in 1963 and is called the Euclid–Mullin sequence.
Further terms in this sequence are on the OEIS webpage https://oeis.org/A000945: 11
is the 12th term, 17 is the 13th term, 19 is the 36th term, and 23 is the 25th term. Mullin
asked if every prime number actually appears as some term in this sequence; this is an
unsolved problem.
Mullin also considered a similar algorithm to generate new primes by using the largest
prime factor of p1 · · · pr + 1 at each step. This “second Euclid–Mullin sequence” starts off
as 2, 3, 7, 43, 139, 50207, 340999. If this list is continued, some primes definitely never show
up in it, such as all the primes from 5 to 47 [5]. In fact, infinitely many primes are missing
from this sequence [2], [9].
For positive x, the number of primes less than or equal to x is written as π(x). For
example, π(10) = |{2, 3, 5, 7}| = 4 and π(12.7) = |{2, 3, 5, 7, 11}| = 5. Because there are
infinitely many primes, π(x) → ∞ as x → ∞. Euclid’s proof gives us the following crude
lower bound on the number of primes up to x.
Corollary 2.3. For x ≥ 2, π(x) > log(log x).
Proof. If p1 , . . . , pn are the first n primes, then Euclid’s proof tells us that a prime factor of
p1 · · · pn + 1 is a new prime, so pn+1 ≤ p1 · · · pn + 1. Using this inequality, we can show by
n−1 1−1
induction that pn ≤ 22 for all n ≥ 1: it’s true when n = 1 since 2 ≤ 22 , and if that
upper bound on primes is true for p1 , . . . , pk then
k−1 k−1 k −1
pk+1 ≤ p1 p2 · · · pk + 1 ≤ 2 · 22 · · · 22 + 1 = 21+2+···+2 + 1 = 22 + 1,
2k
and that last number is at most 2 since 21 x + 1 ≤ x for x ≥ 2 (take x = 2 ). 2k
n−1 n−1
That pn ≤ 22 for all n ≥ 1 means the number of primes up to 22 is at least n, so
n−1
π(22 ) ≥ n for n ≥ 1. For x ≥ 2, choose the integer n ≥ 1 such that 2n−1 ≤ log2 x < 2n .
n−1 n−1
Then x ≥ 22 , so π(x) ≥ π(22 ) ≥ n > log2 (log2 x). Since 0 < log 2 < 1, log2 (y) =
(log y)/(log 2) > log y for y > 0, so log2 (log2 x) > log(log x). 
This lower bound on π(x) is extremely weak because the true order of magnitude of π(x)
is x/ log x: the ratio π(x)/(x/ log x) tends to 1 as x → ∞. That is called the Prime Number
Theorem.

3. Euler’s proof
In 1737, Euler [7, Theorem 7] found a proof of the infinitude of the primes that explains
it by the divergence of the harmonic series. This unexpected link between a property of
THE INFINITUDE OF THE PRIMES 3

prime numbers and calculus (infinite series) could be considered the start of the subject of
analytic number theory, which studies properties of Z using the tools of real and complex
analysis. While Euclid’s proof used the fact that each integer greater than 1 has a prime
factor, Euler’s proof will rely on unique factorization in Z+ .
Theorem 3.1. There are infinitely many primes.
Proof. (Euler) For a prime p, the ratio 1/(1 − 1/p) can be expanded into a geometric series:
1 1 1 1 1
= 1 + + 2 + 3 + 4 + ···
1 − 1/p p p p p
We will now multiply together these ratios for different primes. To see what can be obtained,
1 1 1
let’s look at the product of these terms for the primes 2, 3, and 5: 1−1/2 1−1/3 1−1/5 equals
   
1 1 1 1 1 1 1 1 1
1 + + + + ··· 1+ + + + ··· 1+ + + + ··· ,
2 4 8 3 9 27 5 25 125
and if we multiply together one term from each series (this is the distributive law for
multiplying infinite series, and it is valid when multiplying convergent series of positive
numbers) we will get unit fractions 1/n where n is a product of powers of 2, 3, and 5:
1 1 1 1 1 1 1 1 1 1 1 1 1 1
(3.1) 1+ + + + + + + + + + + + + + + ···
2 3 4 5 6 8 9 10 12 15 16 18 20 24
This looks like the harmonic series, but it is missing terms 1/n where n has a prime factor
greater than 5, such as terms 1/7, 1/11, and 1/14. If we multiply by additional factors
1/(1 − 1/p) for more primes p we’ll introduce into (3.1) new terms 1/n where n has prime
factors involving the new primes and the ones we already used (2, 3, and 5). Because of
unique factorization in Z+ , each term 1/n that appears will do so just once. Therefore if
we multiply together 1/(1 − 1/p) for all the primes,1 (3.1) turns into the sum of 1/n over
all positive integers n:
Y 1 X1
(3.2) = ,
p
1 − 1/p n
n≥1

where the product on the left (denoted Π, like Σ for sum) runs over all prime numbers.
Since the harmonic series diverges, (3.2) tells us that the left side can not be a product
of finitely many terms. Therefore there are infinitely many terms in the product, so there
are infinitely many primes. 
By taking logarithms of a product over primes, Euler showed something new about primes
when we sum their reciprocals:
P
Corollary 3.2. The infinite series p 1/p, where p runs over the primes, diverges.
Proof. For N ≥ 2, multiply out
Y 1 Y 1 1 1 1

= 1 + + 2 + 3 + 4 + ... ,
1 − 1/p p p p p
p≤N p≤N

where the product is over primes p up to N , by multiplying together one term from each of
the geometric series on the right. We obtain unit fractions 1/n that include every integer
1An infinite product, like an infinite series, is a limit of partial products: a a a · · · = lim
1 2 3 k→∞ a1 a2 · · · ak .
4 KEITH CONRAD

n ≤ N , since the prime factors of such n are all at most N . We’ll also get unit fractions
1/n for other n. Therefore
X 1 Y 1
(3.3) <
n 1 − 1/p
n≤N p≤N

As N → ∞ the left side of (3.3) tends to ∞ as N does, so the right side ofQ(3.3) tends to
1
∞ too. That isn’t really telling us anything new, since (3.2) already says p 1−1/p = ∞.
We can get something new from (3.3) by taking logarithms of both sides before letting N
tend to ∞: logarithms turn products into sums and log x is increasing, so (3.3) implies
 
X 1 X 1
(3.4) log  ≤ log
n 1 − 1/p
n≤N p≤N

On the right, log(1/(1 − 1/p)) = − log(1 − 1/p). Recall for |x| < 1 that the power series for
− log(1 − x) is n≥1 xn /n = x + x2 /2 + x3 /3 + · · · , so for small x we have − log(1 − x) ≈
P

x + x2 /2. We need a definite inequality here, not just “≈”, and for that increase x2 /2 up
to x2 : for 0 < x ≤ 1/2, verify by calculus that − log(1 − x) < x + x2 (see the graph below).

y = x + x2
x
1/2 1
y = − log(1 − x)

Using x = 1/p for prime p, so x ≤ 1/2, we have − log(1 − 1/p) < 1/p + 1/p2 . Thus
X X 1 1

− log(1 − 1/p) < + .
p p2
p≤N p≤N

Combining this with (3.4),


 
X 1 X1 X 1
(3.5) log  < + .
n p p2
n≤N p≤N p≤N
P
Now let N → ∞: the left side tends to ∞ since n≤N 1/n → ∞ and on the right side
1/p2 has a finite limit as N → ∞ since n≥1 1/n2 converges. Therefore (3.5) implies
P P
P p≤N P
p≤N 1/p → ∞ as N → ∞, so p 1/p = ∞. 

Since n≥1 1/n = ∞ and p 1/p = ∞ while n≥1 1/n2 < ∞, the primes are “more
P P P
dense” among the positive integers than the squares.
We can derive Corollary 2.3 from Euler’s approach to the infinitude of the primes.
Corollary 3.3. For x ≥ 2, π(x) > log(log x).
THE INFINITUDE OF THE PRIMES 5

Proof. In inequality (3.3), on the right side 1/(1 − 1/p) ≤ 2, so p≤N 1/(1 − 1/p) ≤ 2π(N ) .
Q
R N +1
On the left side, using a Riemann sum approximation to 1 dx/x with rectangles of
width 1, we have
X 1 Z N +1 dx
> = log(N + 1).
n 1 x
n≤N

Therefore log(N + 1) < 2π(N ) when N ≥ 2. For x ≥ 2, letting N ≤ x < N + 1 for an integer
N , we get
log x < log(N + 1) < 2π(N ) = 2π(x) < eπ(x) ,
so π(x) > log(log x). 
P P
Although p 1/p = ∞, the divergence is extremely slow. For example, p<108 1/p ≈
P
3.174975. The partial sums p≤N 1/p diverge at the rate log(log N ), which Euler knew:
the last line of his paper where he proved infinitude of the primes [7] says
1 1 1 1 1
+ + + + + etc. = l. l ∞,
2 3 5 7 11
where l was Euler’s notation for natural logarithms.
In a MathOverflow post about the importance of rigor2, the top answer by Richard
Borcherds says “I once got a letter from someone who had overwhelming numerical evidence
that the sum of the reciprocals of primes is slightly bigger than 3 (he may have conjectured
the limit was π). The sum is in fact infinite, but diverges
P so slowly (like log log n) that
one gets no hint of this by computation.” We saw p<108 1/p is slightly above 3. Since
the solution to log(log x) = 4 is a 24-digitP number, a calculator or computer will never give
convincing evidence of the divergence of p 1/p.
Corollary 3.2 was very important historically in number
P theory: it suggests that a way to
prove a set S of primes is infinite is to show the sum p∈S 1/p is ∞. In fact, 100 years after
Euler’s work Dirichlet used this method in 1837 to prove certain arithmetic progressions
contain infinitely many primes.
Theorem 3.4 (Dirichlet). If a and m are relatively prime positive integers, there are in-
finitely many primes in the arithmetic progression {a + mn : n ≥ 0}. Equivalently, there
are infinitely many primes p such that p ≡ a mod m.
For example, the set of primes p with units digit 7, namely p ≡ 7 mod 10, is infinite. It
starts out as
17, 37, 47, 57, 67, 97, 107, 127, 137, 157, 167, 197, 227, 257, 277, . . .
P
The basic idea in Dirichlet’s proof is to show the series p≡a mod m 1/p over primes p is
infinite when (a, m) = 1, so the set of primes p ≡ a mod m is infinite. Some special cases
of Dirichlet’s theorem (such as infinitude of primes p where p ≡ 3 mod 4) can be proved by
elementary techniques, but I am unaware of any elementary method that shows the list of
primes p ≡ 7 mod 10 is infinite.
A famous unsolved problem in number theory is the infinitude of twin primes, which are
prime pairs that differ by 2,Psuch as 3 and 5 or 29 and 31. If T is the set of primes p such that
p + 2 is also prime, might p∈T 1/p be infinite? That would imply there are infinitely many
P
twin primes. Unfortunately, p∈T 1/p is known to converge (which doesn’t contradict the
2See https://mathoverflow.net/questions/37610.
6 KEITH CONRAD

P
possibility that T is infinite: infinite series can converge). The convergence of p∈T 1/p
P
was proved by Brun [3] in 1919. The convergent series B2 := p∈T (1/p + 1/(p + 2)) is
called Brun’s constant. It is known to be less than 2.4 and is expected to be approximately
1.902. Estimating B2 is a difficult problem, and work on this in 1994 led to the discovery
of a bug in an Intel Pentium chip.

4. Erdős’s proof
P
In a footnote to a short paper [6] on divergence of the series p 1/p over the primes,
Erdős gave the following combinatorial proof that there are infinitely many primes.
Theorem 4.1. There are infinitely many primes.
Proof. Suppose there are finitely many primes p1 , p2 , . . . , pr . Each positive integer up to
N can 2
√ be written uniquely as a b where a is a √ positive integer and b is squarefree. Since
a ≤ N , the number of choices for a is at most N . Since b is a product of distinct primes
and there are r primes (including an empty product for b = 1), √ ther number of choices of b
r 2
is 2 . Therefore the number√of possibilities for a b is at most N 2 . There are N positive
integers up to N , so N ≤ N 2r . By algebra, N ≤ 22r . This is a contradiction for large
enough N . 
Corollary 4.2. For every integer N ≥ 2, π(N ) > (log N )/(2 log 2).
This lower bound on π(N ) is an improvement on the lower bound π(x) > log(log x) that
we saw earlier from the proofs of Euclid and Euler.

Proof. Let r = π(N ). Running through the proof of Theorem 4.1 with p1 , . . . , pr being
the set of primes up to N (not the set of all primes), by writing positive
√ integers as a2 b
for squarefree
√ r √ b we find that the number of integers up to N is at most N 2r . Therefore
N ≤ N 2 = N 2π(N ) , so π(N ) ≥ (log2 N )/2 = (log N )/(2 log 2). 

We can improve on the lower bound in Corollary 4.2 by using a result called Bertrand’s
postulate: for each integer n ≥ 2, there is a prime number lying strictly between n and 2n.3
Theorem 4.3. Bertrand’s postulate implies π(x) > log x for all x ≥ 3.
The inequality π(x) > log x is false for e < x < 3, where π(x) = 1 < log x.

Proof. For x ≥ 8, let 2k ≤ x < 2k+1 for an integer k. The k − 1 open intervals (2, 4), (4, 8),
(8, 16), . . . , (2k−1 , 2k ) each contain a prime number and all of them are all less than x, so
π(x) ≥ k − 1. This lower bound on π(x) does not include the prime 2 and for the interval
(4, 8) we counted just one prime in it but there are two primes in it. Therefore
π(x) ≥ (k − 1) + 2 = k + 1 > log2 x > log x
for x ≥ 8. For 3 ≤ x < 5, log x < 2 ≤ π(x) and for 5 ≤ x ≤ 8, log x < 3 ≤ π(x). 

3The label “Bertrand’s postulate” is used for historical reasons and would be better described as
Bertrand’s conjecture. It was proved by Chebyshev [4, Sect. VI] after being conjectured by Bertrand [1,
p. 129] in order to prove the symmetric group Sm has no subgroup with index between 2 and m when
m ≥ 3.
THE INFINITUDE OF THE PRIMES 7

References
[1] J. Bertrand, “Mémoire sur le nombre de valeurs que peut prendre une fonction quand on y permute les
lettres qu’elle renferme,” J. l’École Roy. Polytech. 18 (1845), 123–140. Online at https://books.google.
com/books?id=Tso6AQAAMAAJ&pg=PA123#v=onepage&q&f=false.
[2] A. Booker, “On Mullin’s second sequence of primes,” Integers 12 (2012), 1167–1177.
[3] V. Brun, “La série 1/5+1/7+1/11+1/13+1/17+1/19+1/29+1/31+1/41+1/43+1/59+1/61+. . ., où
les dénominateurs sont nombres premiers jumeaux est convergente ou finie,” Bull. Sci. Math 43 (1919),
100–104, 124–128. Online at https://gallica.bnf.fr/ark:/12148/bpt6k96292009/f104.image.
[4] P. L. Chebyshev, “Mémoire sur les nombres premiers,” J. Math. Pures et Appl. 17 (1852), 366–390.
Online at http://sites.mathdoc.fr/ JMPA/PDF/JMPA 1852 1 17 A19 0.pdf.
[5] C. D. Cox and A. J. van der Poorten, “On a sequence of prime numbers,” J. Austral. Math. Soc. 8
(1968), 571–574.
P
[6] P. Erdős, “Über die Reihe 1/p,” Mathematica (Zutphen) B7 (1938), 1–2. Online at https://users.
renyi.hu/∼p erdos/1938-13.pdf.
[7] L. Euler, “Variae observationes circa series infinitas,” Commentarii academiae scientiarum Petropolitanae
9 (1744), 160–188. Online at http://eulerarchive.maa.org/pages/E072.html.
[8] A. A. Mullin, “Recursive function theory (A modern look at a Euclidean idea),” Bull. Amer. Math. Soc.
69 (1963), 737.
[9] P. Pollack and E. Treviño, “The primes that Euclid forgot,” Amer. Math. Monthly 121 (2014), 433–437.
[10] P. Ribenboim, The Book of Prime Number Records, Springer-Verlag, New York, 1988.

You might also like